ChaseDream
搜索
返回列表 发新帖
00:00:00

Which of the following, if true, provides evidence that most logically completes the argument below?

According to a widely held economic hypothesis, imposing strict environmental regulations reduces economic growth. This hypothesis is undermined by the fact that the states with the strictest environmental regulations also have the highest economic growth. This fact does not show that environmental regulations promote growth, however, since ______.

正确答案: A

更多相关帖子

524

帖子

15

好友

4712

积分

ChaseDream

注册时间
2003-03-17
精华
8
解析
查看: 7768|回复: 7
打印 上一主题 下一主题

GWD一道逻辑题求解释!!!

[复制链接]
跳转到指定楼层
楼主
发表于 2012-5-19 10:03:11 | 只看该作者 回帖奖励 |倒序浏览 |阅读模式
Which of the following, if true, provides evidence that most logically completes the argument below?
According to a widely held economic hypothesis, imposing strict environmental regulations reduces economic growth. This hypothesis is undermined by the fact that the states with the strictest environmental regulations also have the highest economic growth. This fact does not show that environmental regulations promote growth, however, since ______.
A. those states with the strictest environmental regulations invest the most in education and job training
B. even those states that have only moderately strict environmental regulations have higher growth than those with the least-strict regulations
C. many states that are experiencing reduced economic growth are considering weakening their environmental regulations
D. after introducing stricter environmental regulations, many states experienced increased economic growth
E. even those states with very weak environmental regulations have experienced at least some growth
这道题虽然我晓得是选A,可是不知道该怎么解释得通……
求哪位牛人给解答下!感激不尽啊O(∩_∩)O~
收藏收藏 收藏收藏
沙发
发表于 2012-5-19 10:19:21 | 只看该作者
其实这题是要你反驳“ environmental regulations promote growth”这句话,所以看选项的时候发现:B环境保护更严厉的国家比不严厉的国际经济增长快,明显是在支持不是在反驳。C,很多遭受了经济衰退的国家正在考虑削弱环保规制,讲的是经济衰退的国家要干什么,没说削弱环保规制后的效果是增长经济呢还是什么,无关项。D明显在支持,方向不对。E基本没有环保规定的国家也有经济增长,跟要反驳的那句话完全不沾边。
板凳
 楼主| 发表于 2012-5-19 16:57:53 | 只看该作者
恩啊~~~我也是排除选的A!可是想不通这个A是怎么削弱了呢……
地板
发表于 2012-5-19 20:45:28 | 只看该作者
This is a  weakening type question.

Common sense: Investment in Education and Job Training promotes growth in Economy.

A) simply proclaims that there is another reason to explain the economic growth. Whether investment in eduation and job training would lead to economic growth is not proven right or wrong. But it is a possible reason. That is enough to cast an unfavorable light on the hypothesis the author is trying to undermine.

There is another facet of the problem one needs to consider: the main conclusion.

The author agrees with the widely held hypothesis and he is trying to refute an undermining evidence (most strict regulation is associated with the highest growth.) His conclusion is that "This fact does not show that environmental regulations promote growth," and your job is to find a premise which supports this main conclusion.
5#
 楼主| 发表于 2012-5-20 09:44:16 | 只看该作者
Thx very much! Your explaination is so excellent!
6#
发表于 2012-7-15 11:22:45 | 只看该作者
这道题为什么不是C呢,我觉得c是因果倒置法,削弱了原题啊,题目中说environment regulation导致economic growth,而c中说因为reduced economic growth导致weak regulation
7#
发表于 2013-12-9 20:24:03 | 只看该作者
C是无关项,C并未说states consider weakening their environmental regulations是因为他们 experience reduced economic growth,C只在列举一个fact,还是原文没提过的fact。
8#
发表于 2013-12-9 20:47:56 | 只看该作者
突然想明白,C太忽略了作者的意思了。
作者说:According to a widely held economic hypothesis, imposing strict environmental regulations reduces economic growth. This hypothesis is undermined ....
可是C的意思不就是支持红色部分吗?
说明做complete题和削弱题的区别时,填空部分还是要忠实原文的。
您需要登录后才可以回帖 登录 | 立即注册

Mark一下! 看一下! 顶楼主! 感谢分享! 快速回复:

手机版|ChaseDream|GMT+8, 2024-9-20 02:54
京公网安备11010202008513号 京ICP证101109号 京ICP备12012021号

ChaseDream 论坛

© 2003-2023 ChaseDream.com. All Rights Reserved.

返回顶部